Μονόλογος

Συντονιστές: AΝΔΡΕΑΣ ΒΑΡΒΕΡΑΚΗΣ, silouan, rek2

Άβαταρ μέλους
KARKAR
Δημοσιεύσεις: 15019
Εγγραφή: Τετ Δεκ 08, 2010 6:18 pm

Μονόλογος

#1

Μη αναγνωσμένη δημοσίευση από KARKAR » Σάβ Ιαν 14, 2017 8:09 pm

Μονόλογος.png
Μονόλογος.png (18.22 KiB) Προβλήθηκε 550 φορές
Στην πλευρά AB του ισοσκελούς τριγώνου ABC,(AB=AC) , παίρνουμε σημείο S ,

ώστε : \dfrac{AS}{AB}=\lambda , με 0<\lambda<\dfrac{1}{2} . Ο κύκλος ο οποίος διέρχεται από την κορυφή A , το

σημείο S και το περίκεντρο O του τριγώνου , τέμνει την πλευρά AC στο σημείο P , ενώ

η ευθεία SP τέμνει την προέκταση της BC στο σημείο Q . Υπολογίστε το λόγο : \dfrac{QC}{BC} .

Επαληθεύστε το αποτέλεσμά σας για \lambda=\dfrac{1}{3} .



Λέξεις Κλειδιά:
Άβαταρ μέλους
Doloros
Επιμελητής
Δημοσιεύσεις: 9853
Εγγραφή: Τρί Αύγ 07, 2012 4:09 am
Τοποθεσία: Ιεράπετρα Κρήτης

Re: Μονόλογος

#2

Μη αναγνωσμένη δημοσίευση από Doloros » Σάβ Ιαν 14, 2017 10:07 pm

KARKAR έγραψε:Μονόλογος.pngΣτην πλευρά AB του ισοσκελούς τριγώνου ABC,(AB=AC) , παίρνουμε σημείο S ,

ώστε : \dfrac{AS}{AB}=\lambda , με 0<\lambda<\dfrac{1}{2} . Ο κύκλος ο οποίος διέρχεται από την κορυφή A , το

σημείο S και το περίκεντρο O του τριγώνου , τέμνει την πλευρά AC στο σημείο P , ενώ

η ευθεία SP τέμνει την προέκταση της BC στο σημείο Q . Υπολογίστε το λόγο : \dfrac{QC}{BC} .

Επαληθεύστε το αποτέλεσμά σας για \lambda=\dfrac{1}{3} .


Αν D,E οι προβολές του O στις AC,AD θα είναι OS = OP\,\,\kappa \alpha \iota \,\,OE = OD άρα

\vartriangle ESO = \vartriangle DPO \Rightarrow \boxed{ES = PD} , οπότε

AS + AP = AS + SE + AP - PD = AE + AD = AB = AC .

Με συνέπεια : \boxed{AS = PC\,\,\,\kappa \alpha \iota \,\,SB = AP}
Μονόλογος.png
Μονόλογος.png (32.14 KiB) Προβλήθηκε 524 φορές
Επειδή \dfrac{{AS}}{{AB}} = \lambda  \Rightarrow \boxed{\frac{{AS}}{{SB}} = \frac{\lambda }{{1 - \lambda }}} θα είναι Με εφαρμογή Θ. Μενελάου στο \vartriangle ABC

και τέμνουσα \overline {SPQ} προκύπτει : \dfrac{{AS}}{{SB}} \cdot \dfrac{{BQ}}{{QC}} \cdot \dfrac{{CP}}{{PA}} = 1 \Rightarrow \dfrac{{QC}}{{QB}} = \dfrac{{{\lambda ^2}}}{{{{(1 - \lambda )}^2}}} και άρα

\boxed{\dfrac{{QC}}{{BC}} = \dfrac{{{\lambda ^2}}}{{1 - 2\lambda }}}


Φιλικά, Νίκος


Άβαταρ μέλους
george visvikis
Επιμελητής
Δημοσιεύσεις: 13277
Εγγραφή: Παρ Νοέμ 01, 2013 9:35 am

Re: Μονόλογος

#3

Μη αναγνωσμένη δημοσίευση από george visvikis » Κυρ Ιαν 15, 2017 10:07 am

KARKAR έγραψε:Μονόλογος.pngΣτην πλευρά AB του ισοσκελούς τριγώνου ABC,(AB=AC) , παίρνουμε σημείο S ,

ώστε : \dfrac{AS}{AB}=\lambda , με 0<\lambda<\dfrac{1}{2} . Ο κύκλος ο οποίος διέρχεται από την κορυφή A , το

σημείο S και το περίκεντρο O του τριγώνου , τέμνει την πλευρά AC στο σημείο P , ενώ

η ευθεία SP τέμνει την προέκταση της BC στο σημείο Q . Υπολογίστε το λόγο : \dfrac{QC}{BC} .

Επαληθεύστε το αποτέλεσμά σας για \lambda=\dfrac{1}{3} .
Καλημέρα στους φίλους!
Μονόλογος..png
Μονόλογος..png (17.63 KiB) Προβλήθηκε 492 φορές
Προφανώς τα τρίγωνα AOS, COP είναι ίσα, οπότε AS=PC, SB=AP. Έστω CE||AB.

\displaystyle{CE||AS \Leftrightarrow \frac{{CP}}{{PA}} = \frac{{CE}}{{AS}} \Leftrightarrow \frac{\lambda }{{1 - \lambda }} = \frac{{CE}}{{\lambda b}} \Leftrightarrow } \boxed{CE = \frac{{\lambda^2 {b}}}{{1 - \lambda }}} (1)

\displaystyle{CE||SB \Leftrightarrow \frac{{QC}}{{QB}} = \frac{{CE}}{{SB}}\mathop  \Leftrightarrow \limits^{(1)} \frac{{QC}}{{QB}} = \frac{{\frac{{{\lambda ^2}b}}{{1 - \lambda }}}}{{b(1 - \lambda )}} = \frac{{{\lambda ^2}}}{{{{(1 - \lambda )}^2}}} \Leftrightarrow } \boxed{\frac{{QC}}{{BC}} = \frac{{{\lambda ^2}}}{{1 - 2\lambda }}}

Τέλος για \displaystyle{\lambda  = \frac{1}{3} \Rightarrow }\displaystyle{\frac{{QC}}{{BC}} = \frac{1}{3}}


Απάντηση

Επιστροφή σε “Γεωμετρία - Προχωρημένο Επίπεδο (Juniors)”

Μέλη σε σύνδεση

Μέλη σε αυτήν τη Δ. Συζήτηση: Δεν υπάρχουν εγγεγραμμένα μέλη και 6 επισκέπτες